You are on page 1of 118

PUZZLES - PRACTICE BOOK

Directions (1-5) : Study the information and answer


the following questions:
A, B, C, D, E. G, and I are 7 friends who study in 3 different
standards namely 5th, 6th, and 7th, such that not less than 2
friends study in the same standard.
Each friend has a different favorite Subject namely History,
Civics, English, Marathi, Hindi, Maths and Economics but
not necessarily in the same order.
A likes Math and studies in the 5th standard with only one
other friend who likes Marathi. I studies with two other
friends. Both the friends who study with I like languages
(here languages include only Hindi. Marathi and English. D
Studies in the 6th standard with only one person and does
not like Civics. E studies with only one friend. The one who
likes history does not study in 5th or 6th standard. E does
not like languages. C does not like English, Hindi or Civics.
1. Which combination represents Es favourite subject and
the standard in which he studies?
1) Civics and 7th
2) Economics and 5th
3) Civics and 6th
4) History and 7th
5) Economics and 7th
2. Which of the following is Is favorite subject?
1) History
2) Civics
3) Marathi
4) Either English or Marathi
5) Either English or Hindi
3. Which amongst the following studies in the 7th standard?
1) G
2) C
3) E
4) D
5) Either D or B
4. Which of the following combinations is definitely correct?
1) I and Hindi
2) G and English
3) C and Marathi.
4) B and Hindi
5) E and Economics
5. Which of the following subjects does G like?
1) Either Maths or Marathi
2) Either Hindi or English
3) Either Hindi or Civics
4) Either Hindi or Marathi
5) Either Civics or Economics

Directions (6 - 10): Study the following information carefully


and answer the given questions.
Raju, Ram, Rita and Reena are 4 students of different
classroom coaching centres of ABC, a leading classroom
coaching centre for IAS aspirants.
Each belongs to a different state, ie UP, Uttarakhand, Bihar
and Odisha, but not necessarily in the same order.
ABC arranged an All India Mock-Test series for all the
classroom coaching centres students of ABC in 4 cities
Bangalore, Kolkata, Dehradun and Chennai.
The students Raju, Ram, Rita and Reena appeared for the
test series in different cities. When the results were declared
they got first to fourth position in All-India ranking. They
were students of ABC, ie Lucknow, Delhi, Patna and
Cuttack, but not necessarily in the same order.
The one who appeared at the Bangalore test centre got the
4th position in All-India ranking. But he is not Ram. The
one who appeared at Chennai for the test got the first
position. The one who belongs to Uttarakhand got the third
position, while the one who studied at the Lucknow
centre got the second position in All-India ranking.
Raju and Ram studied neither at the Lucknow centre not at
Delhi. They appeared for test neither at Kolkata nor at
Dehradun. They belong neither to Odisha nor to
Uttarakhand.
The one who studied at the Patna centre does not belong to
Bihar. The one who belongs to Uttarakhand is not Reena.
The one who studied in Delhi did not appear for the test at
Kolkata centre. The one who belongs to UP did not appear
at Bangalore centre.
6. In which of the following cities did Raju appear for the
test?
1) Bangalore
2) Dehradun
3) Chennai
4) Kolkata
5) Data inadequate
7. Which of the following combinations is definitely true in
respect of the given information?
1) Ram Dehradun 2
2) Rita Bangalore 4
3) Reena Kolkata 3
4) Raju Chennai 1
5) None is true
8. Ram belongs to which of the following states?
1) UP
2) Bihar
3) Odisha
4) Cant say
5) None of these
9. Who got the second position in the All-India ranking?
1) Ram
2) Rita
3) Raju
4) Reena
5) Cant say
10. Which of the following information is not true regarding
Rita?
1) Rita got 3rd position in All-India ranking.
2) Rita belongs to UP.
3) Rita appeared for the test at the Dehradun centre.
4) She was a student of Delhi centre.
5) None of these

Solution :
Directions (1-7) Study the information carefully and answer the question that follows
A, B, C, D, E, F, G and H are eight sportspersons, each of them playing a different
game, viz Cricket, Football, Golf, Hockey, Chess, Baseball, Badminton and Tennis (not
necessarily in the same order). All of them are seated around a circular table facing the
centre. A, who plays Golf, sits third to the left of E. The one who plays Baseball sits
second to the right of D. D, who plays Badminton, sits second to the right of B, who
cannot sit adjacent to E. C, who plays Football, sits between the players of Chess and
Baseball. G is a Tennis player, and sits second to the left of H, who is a Cricketer.

1.Who among the following pairs represent immediate neighbours of the Badminton
player?

1) GA

2) AH

3) BH

4) AE

5) None of these

2.Which of the following is definitely true in the context of F?

1) F sits second to the left of the Cricketer.

2) F sits third to the right of A.

3) F is a Hockey player.

4) The one who plays Golf is not an immediate neighbour of F.

5) None of these

3.What is the position of B with respect to the Chess player?

1) Fourth to the right

2) Third to the left

3) Second to the right

4) Fourth to the left


5) Second to the left

4.How many persons are sitting between A and the one who plays Hockey, if counted
clockwise from A?

1) One

2) Two

3) Three

4) Four

5) Six

5.Which of the following games is played by the one who sits second to the left of the
Baseball player?

1) Chess

2) Tennis

3) Cricket

4) Football

5) None of these

6.The one who sits between the Tennis player and the cricketer plays which of the
following games?

1) Hockey

2) Golf

3) Badminton
4) Chess

5) None of these

7.In which of the following pairs the player of first game (of the pair) sits on the
immediate left of the player of the second game (of the pair)?

1) Tennis-Football

2) Cricket-Hockey

3) Baseball-Golf

4) Football-Chess

5) Chess-Golf Directions

(Q. 8-11): Read the following information carefully to answer the given questions.

A university organised exams for six different subjects, viz Maths, Physics, Chemistry,
Electronics, Statistics and English on six days of a week, not necessarily in the same
order.

The exams start from Monday, with a holiday on any day of the week. One full day is
devoted to one exam. The exam of Maths is scheduled immediately after the exam of
Physics. The exam of Electronics is scheduled on Wednesday but not after the exam of
English. The exam of Chemistry is scheduled on Friday. There is only one exam
between the exams of Statistics and Maths. There is only one day when no paper is
scheduled but that is not Saturday. The exam of English is scheduled just before the
holiday.

8.On which of the following days is the exam of Statistics scheduled?

1) Tuesday

2) Wednesday

3) Friday
4) Saturday

5) None of these

9.On which day is a holiday?

1) Sunday

2) Monday

3) Tuesday

4) Wednesday

5) None of these

10.How many exams is/are scheduled between the exams of Maths and Electronics?

1) One

2) Two

3) Three

4) Four

5) None of these

11.Which two exams are scheduled on the first and last day?

1) Electronics, English

2) Maths, English

3) Physics, Chemistry
4) Physics, English

5) None of these

12.Which of the following combinations is correct?

1) EnglishThursday

2) MathsMonday

3) StatisticsSaturday

4) PhysicsMonday

5) None of these

Q1-7

1. 2
2. 5
3. 3
4. 2
5. 5
6. 1
7. 4
Q 8-12

8) 5

9) 1

10) 5

11) 4 12) 4

Directions (Q.1 - 8): Study the following information


carefully to answer the given questions.

Eight persons from different Public Sector Banks, viz UCO


Bank. Syndicate Bank, Canara Bank, PNB, Dena Bank,
Oriental Bank of Commerce, Indian Bank and Bank of
Maharashtra, are sitting in 2 parallel rows containing 4
people each, in such a way that there is an equal distance
between adjacent persons.
In row 1 A, B, C and D are seated and all of them are facing
south. In row 2, P, Q, R and S are seated and all of them are
facing north. Therefore, in the given seating arrangement
each member seated in a row faces another member of the
other row. (All the information given above does not
necessarily represent the order of seating as in the
final arrangement.)
C sits second to right of the person from Bank of
Maharashtra. R is an immediate neighbour of the person
who faces the person from Bank of Maharashtra.
Only one person sits between R and the person from PNB.
The immediate neighbour of the person from PNB faces the
person from Canara Bank.
The person from UCO bank faces the person from Oriental
Bank of Commerce. R is not from Oriental Bank of
Commerce. P is not from PNB. P does not face the person
from Bank of Maharashtra.
Q faces the person from Dena Bank. The one who faces S
sits to the immediate left of A.
B does not sit at any of the extreme ends of the line. The
person from Bank of Maharashtra does not face the person
from Syndicate Bank.
1. Which of the following is true regarding A?
1) The person from UCO Bank faces A.
2) The person from Bank of Maharashtra is an immediate
neighbour of A.
3) A faces the person who sits second to the right of R.
4) A is from Oriental Bank of Commerce.
5) A sits at one of the extreme ends of the line.
2. Who is seated between R and the person from PNB?
1) The person from Oriental Bank of Commerce
2) P
3) Q
4) The person from Syndicate Bank
5) S
3. Who amongst the following sit at extreme ends of the rows?
1) D and the person from PNB
2) The persons from Indian Bank and UCO Bank
3) The persons from Dena Bank and P
4) The persons from Syndicate Bank and D
5) C, Q
4. Who amongst the following faces the person from Bank of
Maharashtra?
1) The person from Indian Bank
2) P
3) R
4) The person from Syndicate Bank
5) The person from Canara Bank
5. P is related to Dena Bank in the same way as B is related to
PNB based on the given arrangement. Who amongst the
following is D related to, following the same pattern?
1) Syndicate Bank
2) Canara Bank
3) Bank of Maharashtra
4) Indian Bank
5) Oriental Bank of Commerce
6. Four of the following five are alike in a certain way based
on the given seating arrangement and thus form a group.
Which is the one that does not belong to that group ?
1) Canara Bank
2) R
3) Syndicate Bank
4) Q
5) Oriental Bank of Commerce
7. Who amongst the following is from Syndicate Bank ?
1) C
2) R
3) P
4) D
5) A
8. C is from which of the following banks ?
1) Dena Bank
2) Oriental Bank of Commerce
3) UCO Bank
4) Syndicate Bank
5) Canara Bank
Directions (Q. 9 - 13): Study the following information
carefully and answer the questions given below.
A group of eight friends O, N, Q, P, M, R, T and S are
sitting in a straight line facing north.
Each of them has different professions Doctor, Banker,
Businessman, Teacher, Lawyer, Engineer, Pilot and CA.
Each of them likes different colours Pink, Yellow, Red,
Black, Blue, Orange, White and Green, but not necessarily
in the same order.
O, who is a Lawyer, sits third to the left of R. Neither O nor
R sits at the extreme ends of the line. M, who likes Pink
colour, is a Businessman. M is not an immediate neighbour
of either O or R. S is two places away from Q and likes
Orange colour. Q, who is a Pilot, likes Red colour. T, who
is a Doctor, sits at an extreme end of the line and likes White
colour. N, who is a Teacher, likes Green colour and sits on
the immediate left of O. O does not like either Blue or
Yellow colour. One who is a CA sits on the immediate left
of the Doctor. R, who is a Banker, does not like Yellow
colour.
9. What is the profession of P?
1) Lawyer
2) Pilot
3) Engineer
4) CA
5) None of these
10. Who sits second to the left of Q?
1) N
2) O
3) R
4) S
5) None of these
11. If N is related to Engineer, O is related to Pilot, Q
is related to which of the following?
1) Doctor
2) Lawyer
3) CA
4) Teacher
5) None of these
12. Who sits at the extreme ends of the line?
1) NT
2) OP
3) MS
4) PR
5) None of these
13. Four of the following five are alike in a certain way based
on their position. Which one does not belong to the group?
1) MN
2) PQ
3) ST
4) PO
5) QR
Solution :

Directions (Q. 1 8): Study the following information carefully to answer the given questions.
Eight persons from different companies, viz. Accenture, Google, Facebook, IBM, Infosys, Microsoft, TCS
and Wipro, are sitting in two parallel rows containing four persons each, in such a way that there is an
equal distance between adjacent persons.
In row 1, J, K, L and M are sitting and all of them are facing north. In row 2, A, B, C and D are sitting
and all of them are facing south.
Therefore, in the given seating arrangement each member sitting in a row faces another member of the
other row. (All the information given above does not necessarily represent the order of seating as in
the final arrangement.)
The person from Infosys faces the one who is on the immediate left of L. L is neither from Google nor
from Facebook.
An immediate neighbour of A faces the person from Accenture. The person from Microsoft faces the
person who is on the left of the person from Google.
There is only are person sitting between the persons from Google and TCS but that person is not J. The
persons from Google and Facebook are not sitting at the extreme ends.
B sits on the immediate left of the person from Infosys. Persons from IBM and Microsoft are immediate
neighbours. C and K are not sitting at any of the ends.
M faces the one who is sitting on the immediate right of the person from Microsoft. A is not from Microsoft
or IBM.
1. Who amongst the following is from Wipro?
1) A
2) L
3) B
4) M
5) Cant be determined
2. Which of the following statements is false regarding M?
1) M is from TCS.
2) M is sitting at one of the extreme ends.
3) M is on the immediate left of the person who is from Facebook.
4) M is sitting opposite the person from Microsoft.
5) All are true
3. Four of the following five are alike in a certain way based on the given seating arrangement and
thus form a group. Which is the one that does not belong to that group?
1) C
2) J
3) A
4) K
5) M
4. L is from which of the following organisations?
1) TCS
2) Accenture
3) IBM
4) Microsoft
5) None of these
5. Who is sitting between D and the person from Infosys?
1) The person who is from Facebook
2) B
3) M
4) The person who faces the one who is from Facebook
5) None of these
6. D is related to Google in the same way as K is related to Wipro based on the given arrangement.
Who amongst the following is L related to, following the same pattern?
1) TCS
2) Microsoft
3) Infosys
4) IBM
5) None of these
7. Who amongst the following faces the one from Microsoft?
1) The person who is from Google
2) L
3) M
4) The person who is from Wipro
5) K
8. Who amongst the following sit on extreme ends of the rows?
1) B and the person from Facebook
2) The persons from TCS and Infosys
3) The persons from Accenture and Microsoft
4) M and the person from Wipro
5) D and T
Directions (Q. 9 11): Study the following information carefully to answer the given questions:
Each of the six friends A, B, C, D, E and F scored different marks in an examination. C scored more
than only A and E. D scored less than only B. E did not score the least. The one who scored the third
highest marks scored 81 marks. E scored 62 marks.
9. Which of the following could possibly be Cs score?
1) 70
2) 94
3) 86
4) 61
5) 81
10. Which of the following is true with respect to the given information?
1) Ds score was definitely less than 60.
2) F scored the maximum marks.
3) Only two people scored more than C.
4) There is a possibility that B scored 79 marks.
5) None is true
11. The person who scored the maximum, scored 13 marks more than Fs marks. Which of
the following can be Ds score?
1) 94
2) 60
3) 89
4) 78
5) 81
ANSWERS:

1. 3
2. 4
3. 5
4. 2
5. 4
6. 2
7. 5
8. 4
9. 1
10. 5
11. 3

Directions (Q. 1-6): Study the following information to


answer the given questions.
Twelve people are sitting in two parallel rows containing six
people each such that they are equidistant from each other.
In row 1, A, B, C, D, E and F are sitting facing South. In row
2, P, Q, R, S, T and V are sitting facing North. Therefore, in
the given seating arrangement, each member sitting in a row
faces another member of the other row.
Three persons sit between C and D. Either C or D sits at an
extreme end of the line.
The one who faces D sits third to the left of R. S faces the
one who sits third to the left of A and he cannot sit adjacent
to R. The immediate neighbour of Q faces the immediate
neighbour of A. Only one person sits between P and T, who
is facing the one sitting on the immediate right of E. Neither
E nor F faces R. V and Q cannot sit adjacent to each other.
1. Who among the following faces B?
1) P
2) Q
3) T
4) R
5) S
2. Who among the following sit at the extreme ends of the
rows?
1) D, Q
2) T, F
3) E, S
4) B, T
5) F, Q
3. If E is related to P in the same way as C is related to S,
which of the following is A related to, following the same
pattern?
1) V
2) R
3) Q
4) T
5) Cant be determined
4. How many persons are sitting between E and F?
1) None
2) One
3) Two
4) Three
5) Four
5. Four of the following five are alike in a certain way based
on the given arrangement and so form a group. Which is the
one that does not belong to that group?
1) PD
2) QF
3) QA
4) SB
5) TE
6. Which of the following is true regarding P?
1) Q is an immediate neighbour of P.
2) P faces the immediate neighbour of B.
3) P sits second from the left end of the row.
4) P sits at one of the extreme ends.
5) P faces the one who is second from the right end of the row.

Directions (Q. 7 - 12): Study the following information


carefully and answer the given questions.
A, B, C, D, E, F, G and H are sitting around a circular
table, facing the centre. Each of them plays a different
game, viz Tennis, Hockey, Cricket, Football, Chess, Golf,
Swimming and Shooting.
The Golfer sits third to the left of A. The one who plays
Chess is an immediate neighbour of A. E, who plays
Hockey, sits third to the right of the Chess player. G is
neither a Swimmer nor a Shooter. B and F always sit
adjacent to each other. There is only one person between A
and E.
The Swimmer sits second to the left of C. F sits second to
the right of the Shooter. Neither B nor F plays Chess. A is
neither a Cricketer nor a shooter and sits opposite D. The
Football player and the Hockey player sit opposite each
other. C plays neither Football nor Golf. There is at least
one player between the Shooter and the Hockey player.
7. Who among the following is a cricketer?
1) A
2) D
3) C
4) G
5) Cant be determined
8. What is the position of H with respect to the golfer?
1) Third to the left
2) Third to the right
3) Fifth to the left
4) Fourth to the left
5) None of these
9. Which of the following is true with respect to the given
sitting arrangement?
1)The Shooter is on the immediate left of the Cricketer.
2)The Golfer and the Swimmer sit opposite to each other.
3)The Football player sits exactly between the Cricketer and the
Chess player.
4)F is a Golfer.
5)None of these
10. Which of the following would come in place of question
mark based upon the given sitting arrangement? EC, BF,
HE, FG, ?
1) CD
2) DB
3) HE
4) BD
5) None of these
11.How many players sit between G and the Shooter when
counted in clockwise direction?
1) Four
2) Three
3) Two
4) One
5) None of these
12.What is the position of the Swimmer with respect to the
Tennis player?
1) Third to the left
2) Immediate left
3) Immediate right
4) Second to the right
5) None

Solution :

1. 4
2. 2
3. 2
4. 4; Three persons A, B and C
5. 3; In all other pairs, either of the persons sits at an extreme
end.
6. 2
7. 3
8. 4
9. 2; Golfer B and Swimmer H sit opposite each other.
10. 2
11. 3; B and F, when counted clockwise.
12. 3
Study the following information carefully and answer the questions given below:

Seven persons namely Paramjit, Tarun, Morya, Jeeva, Vaani, Ram and Waqar are good
friends and are studying in M.Com, M.A. and M.Sc courses. Three are doing M.Com, two are in M.A.
and another two are in M.Sc. Each of them has a very distinct and favorite color choice ranging from
blue, red, yellow, white, black, pink and brown but not necessarily in the same order. None doing
M.Com like either red or black. Morya is doing M.A. and he likes blue. Ram is doing M.Sc and likes
brown. Jeeva is doing M.Com and likes yellow. Paramjit who does not like red is in the same
discipline of Ram. Tarun is in the same discipline of Morya. Vaani does not like pink.

Q1. Which among the following groups is doing M. Com?


a) Jeeva, Vaani and Waqar
b) Vaani, Waqar and Tarun
c) Jeeva, Vaani and Tarun
d) Jeeva, Paramjit and Ram

Q2. What is the color combination choice of those who are in M.Sc discipline?
a) Brown and Pink
b) Black and White
c) Black and Brown
d) Yellow and Black
e) None of The Above

Q3. Which color does Vaani like?


a) Yellow
b) Pink
c) White
d) Brown
e) None of The Above

Q4. What is the color combination choice of those who are in M.A. discipline?
a) Red and Black
b) Blue and Red
c) Blue and Black
d) None of The Above

Q5. Morya is related with which discipline?


a) M.Sc.
b) M.A.
c) M.Com

Solution
Persons Courses Colors
Jeeva M.Com Yellow
Vaani M.Com White
Waqar M.Com Pink
Morya M.A. Blue
Tarun M.A. Red
Ram M.Sc. Brown
Paramjit M.Sc. Black

Answer 1. (Option A)
Answer 2. (Option C)
Answer 3. (Option C)
Answer 4. (Option B)
Answer 5. (Option B)

Directions (1 - 5): Read the following information carefully


and answer the questions that follow.
A, B, C, D, E, F, G and H are eight members of a
family sitting around a circular table and all of them are
facing away from the centre. Each of them has a different
sunsign, viz. Aquarius, Libra, Leo, Cancer, Virgo, Pisces,
Aries and Capricorn, but not necessarily in the same order.
C sits second to the right of Fs wife, whose sunsign is
neither Cancer nor Capricorn. No male is an
immediate neighbour of C. Es son sits second to the left of
H and on the immediate right of the person whose sunsign is
Leo. H, who is brother of B, has the sunsign Capricorn. H is
not an immediate neighbour of Fs wife. Es sons sunsign
is Libra.
Only one person sits between G and H. Fs sister E sits on the
immediate right of her father, whose sunsign is Aries. G is
mother of B. Only one person sits between Fs father and A.
A sits on the immediate left of the person whose sunsign is
Pisces. Only one person sits between F and B. B sits second
to the left of the person whose sunsign is Aquarius. B is
father of D and is not an immediate neighbour of A.
1. Whose sunsign is Aquarius?
1) D
2) G
3) Father of D
4) Cant be determined
5) None of these
2. Who among the following is Es son?
1) B
2) F
3) A
4) D
5) C
3. The person whose sunsign is Virgo is sitting between who
among the following?
1) The persons whose sunsigns are Pisces and Leo
2) The persons whose sunsigns are Aries and Libra
3) The persons whose sunsigns are Libra and Cancer
4) Cant be determined
5) None of these
4. Four of the five are alike in a certain way and hence form
a group. Which is the one that does not belong to that group?
1) D
2) C
3) A
4) F
5) B
5. What is the position of G with respect to her granddaughter
according to the given seating arrangement?
1) 4th to the right
2) 4th to the left
3) Immediate left
4) Immediate right
5) 2nd to the left

Directions (6 - 10): Read the following information carefully


and answer the questions.
Seven people, P, Q, R, S, T, U and V have a seminar, but not
necessarily in the same order, in seven different months (of
the same year), namely January, February, March, June,
August, October and December. Each of them also likes a
different fruit, namely Banana, Grapes, Papaya, Orange,
Mango, Litchi and Apple, but not necessarily in the same
order.
R has a seminar in a month which has less than 31 days. Only
two people have a seminar between R and S. The one who
likes Banana has a seminar immediately before S. Only one
person has a seminar before the one who likes Papaya. Q has
a seminar immediately after the one who likes Papaya.
Only three people have a seminar between Q and the one
who likes Mango. T likes neither Mango nor Papaya. P has
a seminar immediately before T. V likes Apple. The one who
likes Grapes has a seminar in the month which has less than
31 days. The one who has a seminar in March does not like
Orange.
6. In which of the following months does S have a seminar?
1) January
2) Cannot be determined
3) October
4) December
5) June
7. Who among the following have seminars in January and
June respectively?
1) V, S
2) U, S
3) Q, T
4) V, R
5) U, R
8. How many people has/have a seminar between the month
in which V and R have seminars?
1) None
2) Two
3) Three
4) One
5) More than three
9. As per the given arrangement, R is related to Banana and
Q is related to Orange following a certain pattern. Which of
the following is V related to following the same pattern?
1) Mango
2) Litchi
3) Apple
4) Papaya
5) Grapes
10. Which of the following fruits does U like?
1) Orange
2) Papaya
3) Mango
4) Banana
5) Grapes

Solution :
Study the following information carefully and answer the questions given below:

M, K, J, T, R, D and W are seven members of a family. There are two married couples among them
belonging to two different generations. Each of them has a different choice of cuisine - Chinese,
Continental, Thai, Punjabi, South India, Gujarati and Malwani. The grandfather in the family likes
Gujarati food. None of the ladies likes Continental or Thai food. T is the son of M, who likes Chinese
food. W is J's daughter-in-law and she likes South Indian food. K is grandfather of D, who likes
Punjabi food. J is mother of R, who likes Continental food.

1) How is R related to D?
(a) Father (b) Brother (c) Uncle (d) Data inadequate
(e) None of these

2) How many male members are there in the family?


(a) 3 (b) 4 (c) 5 (d) Data inadequate
(e) None of these

3) Which of the following group contains one each from the same generations?
(a) JRT (b) JRW (c) MRD (d) MWT
(e) None of these

4) Which food does T like ?


(a) Gujarati (b) Thai (c) Malwani (d) Data inadequate
(e) None of these

5) Which of the following combinations represents favourite food of the two married ladies ?
(a) Malwani, South Indian (b) South Indian, Punjabi
(c) Punjabi, Malwani (d) Data inadequate
(e) None of these

ANSWERS
1. (c) R is the brother of M, who is father of D. So, R is uncle of D.

2. (d) We can not determine because there is nothing mentioned about D in the question.
3 (a) J belongs to 1st generation, R belongs to 2nd generation and T belongs to 3rd generation.

4 (b) Thai food

5 (a) J likesMalwani and W likes South Indian.

SOLUTION

PERSON CUISINE GENERATION RELATIONSHIPS


K(MALE) GUJARATI 1st FATHER OF M AND
R GRANDFATHER
OF D AND T
J (FEMALE) MALWANI 1st WIFE OF K
M (MALE) CHINESE 2nd SON OF K AND J
W (FEMALE) SOUTH INDIAN 2nd WIFE OF M
R (MALE) CONTINENTAL 2nd BROTHER OF M
D(MALE/FEMALE) PUNJABI 3rd SON/DAUGHTER OF
M &W
T (MALE) THAI 3rd SON OF M
Directions (1 - 5): Study the following information and
answer the given question.
8 persons of different heights are stood in a queue.
Only one person is ahead of the eighth tallest person.
Number of perons ahead of the second tallest is same as
the number of perons behind the second shortest person.
The fifth tallest person stood immediately ahead of the
second tallest. The eigth tallest person is immediately
behind the talest person. The third tallest person is
exactly in between the forth and the sixth tallest person.
Q1. How many persons are ahead of the 5th tallest person.
(1) 6
(2) 7
(3) 5
(4) 2
(5) Cant be determined
Q2. How many persons are ahead of the 3rd tallest person.
(1) 1
(2) 3
(3) 7
(4) 2
(5) Cant be determined

Q3. Who is immediately behind of the first tallest person.


(1) 2
(2) 8
(3) 7
(4) 2
(5) Cant be determined

Q4. How many persons stood in between the 1st tallest and
3rd tallest person.
(1) 2
(2) 8
(3) 7
(4) 2
(5) Cant be determined

Q5. How many persons are ahead of the 3rd tallest person.
(1) 1
(2) 3
(3) 7
(4) 2
(5) Cant be determined

Directions (Q 6 - 12): Study the following information and


answer the given question.
In a Dining Table 8 people Karan, Kirti, Kiran, Kali,
Karm, Kirt, Kush and Kabir from different
cities Kanchipuram, Kannur, Kirtpur Sahib, Kanpur,
Karnal, Kashipur, Kharagpur and Kolhapur not
necessarily in same order sitting around a rectangular
table.
3 persons are sitting on each longer side and each on the
smaller sides.
Kali is sitting second to the right of the person who is
from Kolhapur. Karm is sitting third to the left of the
person who is from Kashipur. Kirt and Kush are sitting
opposite each other.
Kiran is sitting diagonally opposite the person from
Kharagpur. Kirti is sitting opposite the person who is
from Kanchipuram. The person from Kanpur is sitting
second to the right of the person from Karnal and second
to the left of Karan, who is not sitting near the person
who is from Khargpur.
Kush is sitting on the smaller side and to the right of the
person who is from Kharagpur. Persons from Karnal and
Kanpur are not on the same side of the table.
The person from Kanpur is sitting third to the right of the
person from Kolhapur, who is not sitting diagonally
opposite the person from Kannur. The person from
Kanpur is third to the left of Kiran. Kirt sits second to the
left of Kabir. The person from Kannur sits opposite to
the person from Karnal.
Q6. Who is sitting third to the right of the person from
Kannur.
(1) Kiran
(2) Karan
(3) Karm
(4) Karan
(5) None of these

Q7. Who is definitely sitting diagonally opposite to


Karm?
(1) Person from Kanchipuram
(2) Person from Kannur
(3) Person from Kanpur
(4) Person from Kharagpur
(5) Cant be determined

Q8. According to the sitting arrangement what will come


in place of question marks. Kirt : ? :: Kirti : ?
(1) Kirt : Kashipur :: Kirti : Kohlapur
(2) Kirt : Kohlapur :: Kirti : Kirtpur Sahib
(3) Kirt : Karnal :: Kirti : Kohlapur
(4) Kirt : Kharagpur :: Kirti : Kannur
(5) None of these

Q9. From which city does Kush belong?


(1) Kolhapur
(2) Kannur
(3) Kharagpur
(4) Karnal
(5) Kanpur

Q10. Which of the following combinations is definitely


correct?
(1) Kirt Kolhapur
(2) Kabir Karnal
(3) Kali - Kannur
(4) Kirti Kanpur
(5) Kush -Kashipur

Q11. From which city does Karan belong?


(1) Kolhapur
(2) Kannur
(3) Kharagpur
(4) Karnal
(5) Cant be determined

Q12. From which city does Kabir belong?


(1) Kolhapur
(2) Kannur
(3) Kharagpur
(4) Karnal
(5) Cant be determined

Solution :

Ques 1 ; The Arrangement is - Tallest person is in the 1st position.


1. 1
2. 8
3. 5
4. 2
5. 7
6. 4/6
7. 3
8. 6/4
Directions (1 - 5): Study the following information and
answer the given question.
8 friends - J, K, L, M, N, O, P and Q live on eight different
floors of a building but not necessarily in the same order.
The lowermost floor of the building is numbered one, the
one above that is numbered two and so on till the topmost
floor is numbered eight.
J lives on floor numbered 4. Only one person lives
between J and L. O lives on the floor immediately below
L. Only one person lives between O and P. P lives above
O. O lives on floor numbered 5. Only one person live
between K and N, M lives on an even numbered floor but
does not live on floor numbered 8. K lives below Qs
floor.
1. Four of the following five are alike as per the given
arrangement and thus form a group. Which of the following
does not belong to that group?
1) PO
2) QL
3) MJ
4) LJ
5) MN
2. Who lives exactly between the floors on which L and J
live?
1) K
2) P
3) O
4) N
5) M
3. Who amongst the following lives on floor numbered 2?
1) M
2) L
3) O
4) Q
5) P
4. Which of the following is true about M as per given
arrangement?
1) M lives on the lowermost floor.
2) K lives on one of the floor below M.
3) All the given statements are true.
4) Only two people live between M and O.
5) M lives on an odd numbered floor.
5. If L and Q interchange their places and so do M and J,
who amongst the following will live between the floors on
which M and P live, as per the given arrangement?
1) Only Q
2) No one
3) Only L
4) Both P and L
5) Both Q and O

Directions (6 - 10): Study the following information and


answer the given question.
Amit, Arun, Ajay, Arvind, Amar, Amarjeet, Avinash and
Ajit are eight friends studying in engineering colleges in
different streams, viz Aeronautical engineering, computer
engineering, chemical engineering, electronic
engineering, electrical engineering, civil engineering,
mechanical engineering and technical engineering, but not
in the same order.
All of them are seated around a circular table, facing
centre.
Amit sits third to the left of the person who studies
electrical engineering. Only two persons sit between Amar
and Avinash. Neither Amar nor Avinash studies electrical
engineering. The person who studies technical
engineering sits second to the right of Arvind. Arvind is
not an immediate neighbour of Amit. Arvind does not
study electrical engineering and Amit does not study
technical engineering. The one who studies Aeronautical
engineering sits third to the left of Amarjeet.
The one who studies Aeronautical engineering and the one
who studies electrical engineering are not immediate
neighbours. Only one person sits between Arvind and the
one who studies chemical engineering. The person who
studies electronic engineering and the one who studies
computer engineering are immediate neighbours. Only
one person sits between Arun and the one who studies
civil engineering. The one who studies civil engineering
is an immediate neighbour of Ajit. Avinash and Arun are
not immediate neighbours. Arvind does not study
computer engineering.
6. How many people sit between Amar and the one who
studies civil engineering (when counted in clockwise
direction from Amar)?
1) None
2) One
3) Two
4) Four
5) Five
7. Who studies Aeronautical engineering?
1) Arun
2) Ajay
3) Amit
4) Amar
5) Amarjeet
8. Which of the following combinations is definitely true?
1) Arvind-Electrical Engineering
2) Ajay Civil engineering
3) Avinash Chemical engineering
4) Arun Electronic engineering
5) None of these
9. Who studies Chemical engineering?
1) Arun
2) Amarjeet
3) Ajit
4) Amar
5) Can't be determined
10. What is the position of the person who studies
mechanical engineering with respect to Ajay?
1) Immediate left
2) Second to the left
3) Third to the righ
4) Third to the left
5) Second to the right
Solution :

Ques : 1
8Q
7P
6L
5O
4J
3K/N
2M
1 K/N

Ques : 2 Solution

Clock wise ---

Arun Mechanical

Amar chemical

Amarjeet computer

Arvind electronic

Ajit technical

Avinash civil

Amit aeronautic
Directions (1 - 6) Study the following information and
answer the questions given below:
8 friends A, B, C, D, E, F, G and H are sitting around a
circular table not necessarily in the same order. 3 of them
are facing outward while 5 are facing towards the centre.
There are equal number of males and females in the
group.
C is facing the centre. E is sitting third to the right of C. F
is sitting third to the left of E. Three persons are sitting
between F and B. The immediate neighbours of B are
females. G is sitting third to the right of F. D is sitting third
to the right of A.
A is not an immediate neighbour of E.
The immediate neighbours of E are males and are facing
the centre. The immediate neighbours of D are females
and face outside. The one sitting third to the left of B is a
male. No female is an immediate neighbour of G.
Q 1. Who is sitting second to the right of E?
(1) C
(2) B
(3) G
(4) H
(5) None of these
Q 2. How many persons are sitting between H and C when
confused from the left side of H?
(1) One
(2) Two
(3) Three
(4) Four
(5) More than four
Q 3. Which of the following statements are true regarding
H?
(1) The one who is second to the right of H is female
(2) H is facing the centre
(3) H is a male
(4) The immediate neighbors of H is facing outside
(5) None is true
Q 4. What is D's position with respect to G?
(1) Third to the left
(2) Third to the right
(3) Second to the left
(4) Second to the right
(5) None of these
Directions (Q 5 - 6) Four of the following five are alike in a
certain way based on their seating positions in the above
arrangements and hence form a group. Which of the
following does not belong to the group?
Q 5.
(1) BE
(2) CG
(3) GA
(4) DH
(5) AF
Q 6.
(1) B
(2) F
(3)G
(4) A
(5) D
Directions (7 - 13) Study the following information and
answer the questions given below:
Eight friends M, N, O, P, Q, R, S and T are sitting around
circular table and belong to eight different states Uttar
Pradesh, Bihar, Andhra Pradesh, Chandigarh, Punjab,
Haryana, Kerala and Maharashtra. They read three
Newspapers in Hindi, English and Punjabi. At least two
persons read one news paper not necessarily in the same
order.
Only P and Q read Hindi Newspaper. P is third to right of
Q. M is from Kerala reads English Newspaper and is
sitting immediate right of Q. R is from Chandigarh and
reads Punjabi Newspaper. T reads the same Newspaper as
S reads who is third to left of R and fourth to right of N.
O is sitting exact opposite of P who belongs to UP.
Persons belongs to Maharashtra and Andhra Pradesh read
English Newspaper. S is from Haryana while N is from
Andhra Pradesh reads English Newspaper. T is from
Punjab.
7. Who does not read English Newspaper?
(1) O
(2) N
(3) M
(4) T
(5) None of these
8. From which state Q belongs?
(1) UP
(2) Bihar
(3) Maharashtra
(4) Andhra Pradesh
(5) None of these
9. Which of the following is not correct combination?
(1) T-Punjab-Punjabi
(2) Q-Kerala-Hindi
(3) O-Maharashtra-English
(4) P-UP-Hindi
(5) All are correct combinations
10. Which two persons are sitting adjacently who read
Punjabi Newspapers?
(1) RT
(2) RN
(3) RS
(4) TS
(5) None of these
Directions (11 - 13) : Given questions one based on the
above question. One group does not match to group find
that one.
11.
(1) OQ
(2) RN
(3) PT
(4) MS
(5) MO
12.
(1) PQ
(2) RS
(3)TR
(4) OM
(5) SN
13.
(1) NOQ
(2) OQM
(3)QMS
(4) SPT
(5) TNR

Solution :

1-

2.
Directions (1 - 5) Study the following information and
answer the questions given below:
9 people A, B, C, D, E, F, G, H and J stay in a building,
not necessarily in the same order.
The building has 9 floors and only one person stays on one
floor. All of them own one car each, and each car is of a
different colour, i.e. blue, grey, white, black, yellow,
green, red, orange and pink not necessarily in the same
order.
The ground floor is numbered 1, the floor above it, number
2 and so on and the top most floor is numbered 9.
H owns a black coloured car and stays on an even
numbered floor. A stays on any even numbered floor
below the floor on which H stays. The one who owns an
orange coloured car stays on the fourth floor.
E stays on the second floor and owns the white coloured
car. The one who owns a pink coloured car stays on the
third floor. A does not own a green coloured car. There are
two floors between the floors on which the people owning
the red and the black coloured cars stay.
C owns a grey coloured car. There are three floors between
the floors on which C and G stay. D stays on a floor
immediately above J's floor. There is one floor between
the floors on which F and G stay. F does not own the pink
coloured car.
The one who owns the blue car stays on the top-most floor.
F does not stay on the ground floor.
1). Who is staying on the floor no. 7?
1) A
2) B
3) D
4) E
5) F
2). How many persons are staying between D and H?
1) Three
2) Four
3) Two
4) One
5) None of these
3). Who is staying on the second floor?
1) D
2) F
3) E
4) A
5) B or C
4). Who is staying on the ninth floor?
1) G
2) B
3) A
4) C
5) D
5). How many persons are staying between A and C?
1) Three
2) Four
3) Two
4) One
5) None of these

Directions (6 - 10) Study the following information and


answer the questions given below:
A, B, C, D, E, F, G, and H are sitting around a circulr table
facing the centre. Each one of them has a different
profession viz Doctor, Engineer, Architect, Teacher,
Clerk, Shopkeeper, Businessman and Banker.
A sits third to the right of Teacher. D sits second to the left
of G. G is not an immediate neighbour of the Teacher.
Only one person sits between B who is the Shopkeeper
and Teacher.
The one who is an Architect sits third to the right of the
Shopkeeper. H sits between Architect and Engineer. E is
not an immediate neighbour of H.
Engineer sits third to the right of Clerk. Only one person
sits between Businessman and F. E is neither a
Businessman nor a Doctor.
6). Which of the following is true with respect to the given
seating arrangement?
a) E is an architect
b) E is an immediate neighbor of the engineer
c) The clerk is an immediate neighbor of the banker
d) Shopkeeper sits second to the right of the teacher
e) The teacher sits between H and the engineer
7). What is the profession of H?
a) Businessman
b) Banker
c) Architect
d) Teacher
e) Shopkeeper
8). What is the position of doctor with respect to the
banker?
a) Second to the right
b) Third to the left
c) Second to the left
d) Fourth to the left
e) Immediately to the left
9). Who sits exactly between the architect and the
businessman?
a) Banker and shopkeeper
b) C and H
c) Clerk
d) Doctor
e) C and teacher
10). Who amongst the following is a clerk?
a) F
b) D
c) E
d) C
e) G
Solution :

1. 5
2. 1
3. 3
4. 2
5. 2

Floor Person Car


9 B Blue
8 H Black
7 F Green
6 A Yellow
5 G Red
4 D Orange
3 J Pink
2 E White
1 C Grey

6. 3
7. 4
8. 2
9. 4
10. 5
Directions (1 - 6) Study the following information and
answer the questions given below:
Twelve people are sitting in two parallel rows containing
six people each, in such a way that there is an equal
distance between adjacent persons.
In row - 1 P, Q, R, S, T and V are seated and all of them
are facing South. In row - 2 A, B, C, D, E and F are seated
and all of them are facing North. Therefore in the given
seating arrangement each member seated in a row faces
another member of the other row.
S sits third to the right of Q. Either S or Q sits at an extreme
end of the line. The one who faces Q sits second to right
of E. Two people sit between B and F. Neither B nor F sits
at an extreme end of the line. The immediate neighbour of
B faces the person who sits third to left of P. R and T are
immediate neighbours of each other. C sits second to the
left of A. T does not face the immediate neighbour of D.
1. Who amongst the following sit at extreme ends of the
rows?
(1) S, D
(2) Q, A
(3) V, C
(4) P, D
(5) Q, F
2. Who amongst the following faces S?
(1) A
(2) B
(3) C
(4) D
(5) F
3. How many persons are seated between V and R?
(1) One
(2) Two
(3) Three
(4) Four
(5) None
4. P is related to A in the same way as S is related to B based
on the given arrangement. To which of the following is T
related to, following the same pattern?
(1) C
(2) D
(3) F
(4) E
(5) Cannot be determined
5. Which of the following is true regarding T?
(1) F faces T
(2) V is an immediate neighbour of T
(3) F faces the one who is second to right of T
(4) T sits at one of the extreme ends of the line
(5) Q sits second to the right of T
6. Four of the following five are alike in a certain way based
on the given arrangement and so form a group. Which is
the one that does not belong to that group?
(1) A - T
(2) B - T
(3) F - P
(4) C - V
(5) E - Q
Directions (7 - 11): Study the following information
carefully and answer the questions given below:
P, Q, R, S, T, V, W and Z are eight friends studying in
three different engineering colleges - A, B and C in three
disciplines - Mechanical, Electrical and Electronics with
not less than two and not more than three in any college.
Not more than three of them study in any of the three
disciplines.
W studies Electrical in college B with only T, who
studies Mechanical. P and Z do not study in college C
and study in the same discipline but not Electrical.
R studies Mechanical in college C with V, who studies
Electrical. S studies Mechanical and does not study in the
same college where R studies. Q does not study
Electronics.
7. Which of the following combinations of college - student
- specialisation is correct?
1) C-R-Electronics
2) A-Z-Electrical
3) B-W-Electronics
4) B-W-Electrical
5) B-Z-Electronics

8. In which of the following colleges do two students study


in Electrical discipline?
1) A only
2) B only
3) C only
4) Cannot be determined
5) None of these

9. In which discipline does Q study?


1) Electrical
2) Mechanical
3) Electrical or Mechanical
4) Data inadequate
5) None of these

10. In which of the colleges at least one student studies in


Mechanical discipline?
1) A only
2) B only
3) C only
4) Both A and B
5) All A, B and C

11. S studies in which college?


1) A
2) B
3) A or B
4) Data inadequate
5) None of these
Solution :

Answers (1 - 6)
1. (4)
2. (1)
3. (2)
4. (2)
5. (3)
6. (5)

Answers (7 - 11)
7. (4)
8. (3)
9. (1)
10. (5)
11. (1)
Person College Stream
P A Electronics
Q C Electrical
R C Mechanical
S A Mechanical
T B Mechanical
V C Electrical
W B Electrical
Z A Electronics
Directions (1 - 5) Study the following information carefully
and answer the question given below:
In a conference 8 people A, B, C, D, E, F, G and H from
different cities Delhi, Pune, Kochi, Mumbai, Chennai,
Lucknow, Bhatinda and Amritsar not necessarily in same
order are sitting around a rectangular table. 3 persons are
sitting on the each longer side and 2 on the smaller side.
D is sitting second to the right of the person who is from
Amritsar. E is sitting third to the left of the person who is
from Lucknow. F and G are sitting opposite to each
other. C is sitting diagonally opposite to the person from
Bhatinda.
B is sitting opposite to the person who is from Delhi. The
person from Mumbai is sitting second to the right of the
person from Chennai and second to the left of A, who is
not sitting near to the person who is from Bhatinda.
G is sitting on the smaller side and to the right of the
person who is from Bhatinda. Persons from Chennai and
Mumbai are not on the same side of the table. Person
from Mumbai is sitting third to the right of the person
from Amritsar, who is not sitting diagonally opposite to
the person from Pune.
1. Who is sitting third to the right of the person
from Lucknow.
(1) E
(2) Person from Chennai
(3) A
(4) Person from Bhatinda
(5) None of these

2. Who is definitely sitting diagonally opposite to


D?
(1) A
(2) C
(3) Person from Delhi
(4) Person from Kochi
(5) Cant be determined

3. According to the sitting arrangement what will


come in place of question mark? D : Mumbai ::
C:?
(1) Lucknow
(2) Chennai
(3) Kochi
(4) Amritsar
(5) None of these

4. From which city does A belong?


(1) Pune
(2) Delhi
(3) Chennai
(4) Amritsar
(5) Cant be determined

5. Which of the following combinations is


definitely correct?
(1) H Amritsar
(2) D Chennai
(3) APune
(4) GMumbai
(5) None of these
Directions (6 - 10) Study the following information
carefully and answer the question given below:
J, K, L, M, N, O and P are seven different boxes of
different colours i.e. Brown, Orange, Silver, Pink,
Yellow, White and Green but not necessarily in the same
order.
Box which is of Brown
colour is immediately above J.
There are only two box
between M and the box which is of Brown colour.
Box which is of Silver colour is above M but not
immediately above M. Only three box are between L and
the box which is of Silver colour.
The box which is of
Green colour is immediately above L.
The box which is of Pink colour is immediately
above the box G. Only one box is there between K and
N. Box K is above N. Neither box K nor J is of Yellow
colour. J is not of orange colour.
6. How many box is /are there between M and G?
(a) One
(b) Two
(c) Three
(d) Four
(e) None

7. What is the colour of O?


(a) Green
(b) Brown
(c) Silver
(d) Pink
(e) Cant be determined.

8. Find the pair of colour and boxes which is not correct?


(a) K-Pink
(b) O-Silver
(c) J-white
(d) G-Brown
(e) None of these

9.Which of the following condition is correct regarding


yellow colour with respect to N?
(a) There are one person between N and Yellow colour
(b) N is immediately above of yellow colour
(c) Yellow colour related to the person immediately above N
(d) All of the above is true
(e) None of the above is true.

10. Which of the following colour is belong to J?


(a) Brown
(b) Pink
(c) Yellow
(d) Orange
(e) None of these.
Solution :

Ans (1 - 5) City of A, C & E is not confirm, B or H is either


form Amritsar or Chennai.
1. 4
2. 2
3. 1
4. 5
5. 4

6. (b)
7.(c)
8.(a)
9.(c)
10.(e)
Boxes Colours
O Silver
K Orange
M Yellow
N Green
L Pink
G Brown
J White
Directions (1 - 7) Study the following information carefully
and answer the question given below:
Eight persons - H, I, J, K, L, M, N and O - are standing in
a straight line at equidistant. Some of them are facing
north while others are facing south.
M is standing third to the right to H. M is standing at one
of the extreme ends. L is standing third to the left of H.
The immediate neighbours of J face north. N is not an
immediate neighbour of H.
The persons standing at the extreme ends face the same
direction (both are facing either North or South). The
immediate neighbours of H face just opposite direction as
that of M. The immediate neighbours of O face opposite
direction with respect to each other.
One of the immediate neighbour of L is K who is facing
north. I is standing between J and M. Not more than four
persons are facing north.
1. Who among the following is third to the left of N?
(1) K
(2) J
(3) H
(4) I
(5) O
2. The immediate neighbours of L are:
(1) M and N
(2) N and O
(3) K and N
(4) N and H
(5) J and H
3. How many persons are standing exactly between I and
O?
(1) Three
(2) Four
(3) One
(4) Two
(5) None
4. Who among the following is to the immediate left of H?
(1) O
(2) J
(3) I
(4) L
(5) K
5. Four of the following five are alike in a certain way based
on the above arrangement and hence form a group. Which
of the following does not belong to that group?
(1) N
(2) L
(3) O
(4) J
(5) K
6. Who among the following is exactly between L and J?
(1) N
(2) O
(3) H
(4) I
(5) None
7. Who among the following is fourth to the right of J?
(1) N
(2) I
(3) H
(4) L
(5) K
Directions (8-14) Study the following information carefully
and answer the questions that follow
Eight people namely A, B, C, D, E, F, G and H are sitting
in a straight line from East to West. Some are facing North
direction and some are facing South direction.
H sits to the third left of C and faces South. F is sitting at
the one end of the extreme ends and is facing North. B and
G are facing in same direction and person sitting between
them is facing opposite direction. A is sitting third from
left end and is facing South direction. A is sitting adjacent
to E and both are facing in opposite direction.
D is sitting to the immediate right of B and both are facing
in same direction. C is facing in North direction. A and D
are not neighbours. Neighbours of H are facing in same
direction which is opposite to the direction of H is facing.
E sits to the right of A.
8. Who is sitting between A and H?
(1) F
(2) E
(3) G
(4) B
(5) None of these
9. Which of the following pairs has the person facing the
same direction?
(1) E, A
(2) G, H
(3) A, G
(4) H, D
(5) B, D
10. How many persons are facing in North Direction?
(1) 2
(2) 3
(3) 4
(4) 5
(5) 6
11. Who is sitting third from the eastern end of the
arrangement?
(1) E
(2) G
(3) B
(4) C
(5) None of these
12. How many persons are sitting between E and B?
(1) 2
(2) 3
(3) 4
(4) 5
(5) 6
13. Which one of the following is correct?
(1) A sits between G and B
(2) D sits to the second left of H
(3) H and B are facing same direction
(4) Person sitting to the left of A faces same direction
(5) All of the above
14. Who sits 3rd to the left of A?
(1) B
(2) D
(3) H
(4) G
(5) None of these
Solution :

Answers (1 - 7)
1 (2) J
2 (3) K and N
3 (4) Two
4 (1) O
5 (5) K
6 (2) O
7 (4) L

Answers (8 - 14)
8 (3) G
9 (5) B, D
10 (5) 6
11 (3) B
12 (2) 3
13 (2) D sits to the second left of H
14 (1) B
Directions (Q. 1 7): Study the following information
carefully and answer the given questions
A, B, C, D, E, F, G and H are sitting around a circle
facing the centre but not necessarily in the same order.
B sits second to the left of Hs husband. No female is an
immediate neighbour of B. Ds daughter sits second to
the right of F. F is the sister of G. F is not an immediate
neighbour of Hs husband. Only one person sits between
A and F. A is father of G. Hs brother D sits on the
immediate left of Hs mother. Only one person sits
between Hs mother and E. Only one person sits between
H and G. G is the mother of C. G is not an immediate
neighbour of E.
1. What is the position of A with respect to his mother-in-
law?
1) Immediate left
2) Third to the right
3) Third to the left
4) Second to the right
5) Fourth to the left

2. Who amongst the following is Ds daughter?


1) B 2)
C 3) E
4) G
5) H

3. What is the position of A with respect to his


grandchild?
1) Immediate right
2) Third to the right
3) Third to the left
4) Second to the left
5) Fourth to the left

4. How many people sit between G and her uncle?


1) One
2) Two
3) Three
4) Four
5) More than four

5. Four of the following five are alike in a certain way


based on the given information and so form a group.
Which is the one that does not belong to that group?
1) F
2) C
3) E
4) H
5) G

6. Which of the following is true with respect to the given


seating arrangement?
1) C is cousin of E.
2) H and Hs husband are immediate neighbours of each
other.
3) No female is an immediate neighbour of C.
4) H sits third to the left of her daughter.
5) B is mother of H.

7. Who sits on the immediate left of C?


1) Fs grandmother
2) Gs son
3) Ds mother-in-law
4) A
5) G

Directions (Q.8 12): Study the following information


carefully and answer the questions given below.
Eight persons W, L, X, M, A , V, Z and Y are sitting
around a square table facing centre. Four of them sit at
the corners and the other four at the middle of the sides.
Each of them likes different cars Swift, Maruti, Alto,
Zen, Ford, Nano, Santro and Safari. but not necessarily
in the same order.
A sits opposite the person who likes Ford and third to the
left of L. L likes neither Maruti nor Ford. M sits between
L and X, who likes Zen. M does not like Ford. Y, who
likes Santro, sits second to the left of X. V, who likes
Safari, is not an immediate neighbour of Y. The person
who likes Maruti sits on the immediate left of the person
who likes Alto. The person who likes Nano is not an
immediate neighbour of the person who likes Zen. W
does not like Nano.
8. Who sits opposite the person who likes Alto?
1) Y
2) X
3) M
4) W
5) None of these

9. Who sits between the person who likes Safari and the
one who likes Santro?
1) L
2) W
3) X
4) A
5) None of these

10. If W is related to Maruti, and A is related to


Nano, V is related to which of the following?
1) Safari
2) Swift
3) Alto
4) Ford
5) None of these

11. Who sits fourth to the left of A?


1) L
2) V
3) W
4) M
5) None of these

12. Who likes Alto car?


1) Y
2) W
3) V
4) L
5) None of these
Solution

Answer (1 - 7)
1. 4; B is mother-in-law of A, Hence, As position with
respect to his mother-in-law is second to the right.
2. 3
3.1; C is grandchild of A. Thus, As position with respect to
his grand child is immediate right.
4. 3; D is uncle of G.
5. 2; Except C, all are females.
6. 5
7.1

Answer (8 - 12)
8. 1
9. 5; Z
10.3
11.3
12.4
Directions (Q. 1 - 5): Study the information carefully and
answer the following questions:
A building has eight floors from 1 to 8 in such a way that
ground floor is numbered 1, the above floor is numbered
2 and so on. The top most floor is numbered 8.
Eight persons G, P, M, R, Q, A, D and C are staying each
on one of these floors. A is staying on floor 6. There is a
gap of three persons between G and R. C is staying on the
topmost floor. Neither P nor M is staying on the floor 5. Q
is staying middle of P and M. R is staying on the floor 1.
M is not an immediate neighbour of G.
Q1. Who is staying on the floor no. 7?
1) A
2) P
3) D
4) R
5) M
Q2. How many persons are staying between D and P?
1) Three
2) Four
3) Two
4) One
5) None of these
Q3. Who is staying on the second floor?
1) Q
2) M
3) P
4) A
5) Q or M
Q4. If all persons are arranged in alphabetical order such
as A occupies floor 1 then who will occupy floor 5?
1) G
2) Q
3) R
4) M
5) P
Q5. Immediate neighbors of the person who is staying in
floor 3?
1) R and P
2) A and P
3) G and M
4) P and M
5) D and G

Directions (Q. 6 - 10): Study the information carefully and


answer the following questions:
Eight friends Romil, Ramesh, Rakesh, Rohit, Rahul,
Abhijit, Abhishek and Anil are sitting around a circular
table, not necessarily in the same order. Four of them are
facing inside others are facing outside. They are belong to
eight different cities Bhopal, Patna, Kolkata, Delhi,
Gwalior, Bengaluru, Chennai and Rajkot, but not
necessarily in the same order.
Abhijit faces the Centre and sits third to the right of
Rakesh. Rohit belongs to Kolkata and faces the person
who belongs to Bengaluru. Abhishesk sits third to the right
of Ramesh, who stays in Bhopal.
The persons who belong to Delhi and Gwalior are facing
to the same direction (inside or outside). Rahul is sitting
between the person who belongs to Kolkata and the one
from Rajkot respectively. Romil belongs to Gwalior and
Rakesh belongs to Patna.
The person who belongs to Chennai is facing outward and
immediate neighbor of Rajkot. Anil is immediate neighbor
of the persons who belong to Gwalior and Chennai. Rahul
is immediate left of Rohit.
Q6. Who belongs to Bengaluru?
1) Romil
2) Rohit
3) Anil
4) Abhishek
5) Rahul
Q7. Immediate neighbors of Romil?
1) Romesh and Rakesh
2) Rahul and Rohit
3) Anil and Ramesh
4) Abhishek and Rahul
5) Rohit and Abhijeet
Q8. Which of the following pair is true?
1) Romil Bhopal
2) Anil Bengaluru
3) Rohit Delhi
4) Abhijeet Delhi
5) Abhishek Patna
Q9. Position of Ramesh with respect to Rahul?
1) Third to the right
2) Fourth to the left
3) Fifth to the right
4) Second to the right
5) Third to the left
Q10. If Rohit and Anil interchange their positions and
similarly Abhijit and Rakesh interchange their positions
then what is the position of Romil with respect to Rakesh?
1) Third to the left
2) Third to the right
3) Second to the left
4) Immediate left
5) None of these
Solution :

Q1-3
Q2-3
Q3-2
Q4-4
Q5-4
Floor Persons
No.
8 C
7 D
6 A
5 G
4 P
3 Q
2 M
1 R

Q6-3
Q7-3
Q8-2
Q9-5
Q 10 - 1
Directions (1-6): Read the following information carefully
and answer the questions that follow:
Seven friends A, B, C, D, E, F and G are sitting around a
circular table facing either the centre or outside. Each one
of them belongs to a different department viz. Finance,
Marketing, Sales, HR, Corporate Finance, Investment
Banking and Operations but not necessarily in the same
order.
C sits third to the right of G. G faces the centre. Only one
person sits between C and the person working in HR
department. Immediate neighbours of C face outside.
Only one person sits between F and D. Both F and D face
the centre. D does not work in HR or Finance department.
A, works in Investment Banking department and faces the
centre. Two people sit between the persons who work in
Investment Banking and Marketing departments. The
person who works in Corporate Finance sits to the
immediate left of E. C faces same direction as E. The
person who works in Corporate Finance sits to the
immediate right of the person who works for Operations
department.
1. B, works in which of the following departments?
(1) Finance
(2) Marketing
(3) HR
(4) Corporate Finance
(5) Operations
2. What is the position of B with respect to the person who
works sales for Sales department?
(1) Immediate right
(2) Third to the left
(3) Second to the right
(4) Second to the left
(5) Fourth to the right
3. Who sits to the immediate right of E?
(1) The person who works in Marketing department
(2) C
(3) B
(4) The person who works for HR department
(5) A
4. Who amongst the following sits exactly between C and
the person who works for HR department?
(1) B
(2) The person who works for Marketing department
(3) The person who works for Operations department
(4) D
(5) G
5. Who amongst the following sit between the persons who
work for Marketing and Investment Banking departments
when counted for the left hand side of the person working
for Marketing department?
(1) F & G
(2) E & C
(3) C & B
(4) F & D
(5) B & D
6. How many people sit between the person who works for
Operation department and A, when counted from the right
hand side of A?
(1) One
(2) Two
(3) Three
(4) Four
(5) More than four

Directions (7-11): Read the following information carefully


and answer the given questions.
Eight persons Ankita, Babita, Chottu, Dheeru, Ehshan,
Farukh, Golu and Harshita are sitting around a circle
facing towards the centre. There are three generations in
the family and three married couples.
Babita and Golu have two children. Ehshan, the husband
of Ankita is sitting to her immediate right and their child
is sitting exactly opposite to Ankita.
Ankita's brother is sitting 2nd to her left with their father
on his left.
Dheeru's wife Harshita is to her immediate right. Farukh
& Babita are sitting together. Babita is an immediate
neighbor of Ehshan.
7. Who is the child of Ankita?
(1) Babita
(2) Dheeru
(3) Golu
(4) Chottu
(5) Farukh
8. Who is to the immediate left of Golu?
(1) Dheeru
(2) Chottu
(3) Ankita
(4) Farukh
(5) Harshita
9. Which of the following pairs is a bachelor in the family?
(1) Dheeru & Ehshan
(2) Babita & Ankita
(3) Farukh & Golu
(4) Chottu & Farukh
(5) Golu & Dheeru
10. Which of the following has the first person sitting to
immediate left of the second person?
(1) Golu - Chottu
(2) Farukh - Babita
(3) Harshita - Dheeru
(4) Bablu - Ehshan
(5) None of these
11. Who is the child of Harshita?
(1) Ankita
(2) Babita
(3) Dheeru
(4) Farukh
(5) None of these
Solution :

Qs 1 (5)
Qs 2. (4)
Qs 3. (2)
Qs 4. (2)
Qs 5. (4)
Qs 6. (5)

Qs 7 (4)
Qs 8. (2)
Qs 9. (4)
Qs 10. (5)
Qs 11. (4)
Directions (Questions 1 - 5): Study the following
information and answer the following questions.
Eight people E, F, G, H, J, K, L and M are sitting around
a circular table facing the centre. Each of them is of a
different profession Chartered Accountant, Columnist,
Doctor, Engineer, Financial Analyst, Lawyer, Professor
and Scientist but not necessarily in the same order.
F is sitting second to the left of K. The Scientist is an
immediate neighbour of K. There are only three people
between the Scientist and E. Only one person sits between
the Engineer and E. The Columnist is to the immediate
right of the Engineer. M is second to the right of K.
H is the Scientist. G and J are immediate neighbours of
each other. Neither G nor J is an Engineer. The Financial
Analyst is to the immediate left of F. The Lawyer is
second to the right of the Columnist. The Professor is an
immediate neighbour of the Engineer. G is second to the
right of the Chartered Accountant.
1. Who is sitting second to the right of E?
(1) The Lawyer
(2) G
(3) The Engineer
(4) F
(5) K
2. Who amongst the following is the professor?
(1) F
(2) L
(3) M
(4) K
(5) J
3. Four of the following five are alike in a certain way based
on the given arrangement and hence from a group. Which
of the following does not belong to that group?
(1) H - Chartered Accountant
(2) M - Doctor
(3) J - Engineer
(4) Financial Analyst - L
(5) Lawyer - K
4. What is the position of L with respect to the Scientist?
(1) Third to the left
(2) Second to the right
(3) Second to the left
(4) Third to the right
(5) Immediate right
5. Which of the following statements is true according to the
given arrangement?
(1) The lawyer is second to the left of the Doctor
(2) E is an immediate neighbour of the Financial Analyst
(3) H sits exactly between F and the Financial Analyst
(4) Only four people sit between the Columnist and F
(5) All of the given statements are true
Answers (1 - 5)
1 (2)
2 (4)
3 (3)
4 (2)
5 (1)
Directions (Questions 6 - 12): Study the following
information and answer the questions given below:
Eight students - A, B, C, D, E, F, G and H are sitting
around a circular table but not necessarily in the same
order. Three of them facing outward while five are facing
towards the centre. There are equal number of males and
females in the group.
C is facing the centre. E is sitting third to the right of C. F
is sitting third to the left of E. Three persons are sitting
between F and B. The immediate neighbours of B are
females. G is sitting third to the right of F. D is sitting third
to the right of A. A is not an immediate neighbour of E.
The immediate neighbours of E are males and are facing
the centre. The immediate neighbours of D are females
and face outside. The one sitting third to the left of B is a
male. No female is an immediate neighbour of G.
6. Who is sitting second to the right of E?
(1) C
(2) B
(3) G
(4) H
(5) None of these
7. How many students are sitting between H and C when
counted from the left side of H?
(1) One
(2) Two
(3) Three
(4) Four
(5) More than four
8. Which of the following statements is true regarding H?
(1) The one who is second to the right of H is a female.
(2) H is facing the centre
(3) H is a male
(4) The immediate neighbours of H are facing outside
(5) None is true
9. What is the D's position with respect to G?
(1) Third to the left
(2) Third to the right
(3) Second to the left
(4) Second to the right
(5) None of these
For Questions (10-12): Four of the following five are alike
in a certain way based on their seating positions in the
above arrangement and hence form a group. Which of the
following does not belong to the group?
10. (1) BE
(2) CG
(3) GA
(4) DH
(5) AF
11. (1) B
(2) F
(3) G
(4) A
(5) D
12. If all the friends are asked of sit in an alphabetical order
starting from A in an anticlockwise direction, the positions
of how many will remain unchanged (excluding A)?
(1) Four
(2) Three
(3) Two
(4) One
(5) None
Answers (6 - 12)
6 (3)
7 (2)
8 (1)
9 (2)
10 (2)
11 (4)
12 (4)
SET 3
Read the following passage carefully and answer the question given below it.

Six friends Abhi, Deep, Myank, Prince, Rambo and Sumit married within a year in the months
of February, April, July, September, November and December and in the cities of Ahmedabad,
Bengaluru, Chennai, Delhi, Mumbai and Kolkata but not necessarily following the above order. The
brides names were Geetu, Jasmeet, Harman, Bindia, Inpreet and Vanshika, once again not following
any order. The following are some facts about their weddings.
(i) Myanks wedding took place in Chennai, however he was not married to Geetu or Vanshika

(ii) Abhis wedding took place in Ahmedabad and Rambos in Delhi; however neither of them was
married to Jasmeet or Bindia.

(iii) The wedding in Kolkata took place in February

(iv) Harmans wedding took place in April, but not in Ahmedabad

(v) Geetu and Inpreet got married in February and November and in Chennai and Kolkata but not
following the above order

(vi) Prince visited Bengaluru and Kolkata only after his marriage in December

(vii) Sumit was married to Jasmine to September

Q1. Harmans husband is


a) Abhi b) Deep c) Rambo
d) Prince e) Myank

Q2. Deeps wedding took place in


a) Bengaluru b) Mumbai c) Kolkata
d) Delhi e) Chennai

Q3. In Mumbai, the wedding of one of the friends took place in the month of
a) April b) September c) November
d) December e) July

Q4. Sumits wedding was held in


a) Bengaluru b) Chennai c) Kolkata
d) Delhi e) Mumbai

Q5. Geetu was married to ____


a) Mayank b) Deep
c) Princ d) None of these

Solution
Answer 1. Option c)
Answer 2. Option c)
Answer 3. Option d)
Answer 4. Option a)
Answer 5. Option b)

Husband Wife Place Month


Abhi Vanshika Ahmedabad July
Deep Geetu Kolkata February
Myank Inpreet Chennai November
Prince Bindia Mumbai December
Rambo Harman Delhi April
Sumit Jasmeet Bengaluru September

SET 4
Directions (Q. 1-5) Study the following information carefully and answer the questions given
below :-

Five friends Pawan, Qureshi, Rajan, Sultan and Tango are Musician, Architect, Doctor,
Engineer and Artist by profession and like White, Blue, Red, Yellow and Green colour but not
necessarily in that order. Their hobbies are Net Surfing, Gardening, Reading, Painting and Dancing
but not necessarily in the same order.

The person whose hobby is dancing preferred lemonade to cola while others preferred cola to
lemonade in beverages.
The four friends who took cola were Pawan, the one who is an Engineer, the person whose
favouritecolour is Green and the one whose hobby is net surfing.
Sultan did not take lemonade and his favouritecolour is White.
Qureshisfavouritecolour is Blue. He did not like lemonade.
Tangos hobby is not painting, reading or gardening.
Sultan clicks a picture of his friend who is an Engineer.
The person whose favouritecolour is Red likes painting and the person who is artist likes
gardening.
Sultan is not a doctor. The person who is a doctor takes cola. The person who is an Engineer
likes Blue colour.
The musicians favouritecolour is not Yellow. Rajansfavouritecolour is Green.

Q1. Who among the following is a Doctor?


a) Rajan b) Pawan c) Sultan
d) Cant say e) None of these

Q2. Qureshis hobby is


a) Reading b) Painting c) Gardening
d) Cant say e) None of these

Q3. The person who likes Blue colour is a/an


a) Architect b) Musician c) Engineer
d) Cant say e) None of these

Q4. Whose favouritecolour is Yellow?


a) Tango b) Rajan c) The one who is an artist
d) Cant say e) None of these

Q5. Which of the following combinations is not correctly matched?


a) Tango-Architect-Yellow-Dancing-Cola
b) Rajan-Artist-Green-Gardening-Cola
c) Qureshi-Engineer-Blue-Reading-Cola
d) Pawan-Doctor-Red-Painting-Cola
e) None of these

Answers
Q 1) B

Q 2) A

Q 3) C

Q 4) A

Q 5) A

SET 5
Question:-

Arti, Baby, Chandni, Dolly, Esha, Falguni, Gopi and Himani are sitting around a square table in such
a way that four of them sit at four corners of the square while four sit in the middle of each of the four
sides. The one who sits at the four corners face the centre of the table while those who sit in the
middle of the sides faces outside.

Each of them likes a different subject - Mathematics, Hindi, English, Biology, Chemistry, Physics,
History and Geography. (None of the information given is necessarily in the same order)

Chandni sits third to the left of the person who likes Geography.The one who likes
Geography faces outside.There are only two persons sit between Chandni and Himani.
Dolly sits on the immediate left of the one who likes Physics. Gopi does not likes Physics.
Esha likes History. Esha is not immediate neighbour of Arti.
The person who likes Hindi is an immediate neighbour of Esha.
The person who likes Biology is an immediate neighbour of Falguni.
The one who likes Mathematics sits in the immediate right of Himani.The one who likes
Chemistry sits second to the right of Gopi. G is neither an immediate neighbour of Himani
nor Chandni. Gopi does not like Geography.
There is only one person sits between Arti and the one who likes Chemistry.

(A) Who among the following sits diagonally opposite the one who likes Mathematics?
1) The one who likes Hindi
2) Dolly
3) Arti
4) The one who likes English
5) The one who likes Biology

(B) Who among the following represent the immediate neighbours of the one who likes Chemistry?
1) Baby, Falguni
2) Chandni, Esha
3) Baby, Esha
4) Dolly, Falguni
5) Falguni, Himani

(C) Who among the followings select exactly between Himani and Baby?
1) Chandni
2) The one who likes Hindi
3) The one who likes Biology
4) Gopi
5) Arti

(D) Which of the following is true regarding Baby?


1) Baby is one of the immediate neighbours of Dolly
2) The one who likes Geography is an immediate neighbour of Baby
3) Baby sits second to the left of Himani
4) Baby likes History
5) Baby is the immediate neighbour of the one who like Mathematics.

(E) What is the position of the one who likes Physics with respect to Gopi?
1) Second to the Left
2) Third to the right
3) Fourth to the left
4) Second to the right
5) Third to the left

(F) Which of the following subjects does Dolly like?


1) Biology
2) Mathematics.
3) Hindi
4) Chemistry
5) English

(G) Who among the following likes Geography?


1) Baby
2) Faguni
3) Himani
4) Arti
5) Dolly
Reasoning Puzzles - Workbook

ANSWERS
(A) 1
(B) 3
(C) 5
(D) 5
(E) 5
(F) 1
(G) 2

SOLUTION

SET 6
Question:-

Four people were being interviewed for the same job, on the same day but in different rooms(R1, R2,
R3 and R4), at different time and by different interviewers.(First sentence what do you do? Make a
table!)

Determine which candidate was interviewed by whom, at which time and in which room.(What can
be arranged first in the table? What can logically be the first coloumn? Room numbers, because
other information cannot be logically arranged!)

By Ramandeep Singh Page 8


(i) Teenas appointment was just after Mr. Sharmas, which was just after that of the person
in room R2.
(ii) Mr. Narurkars appointment was atleast two hours later than Bimals.
(iii) Mr. Joshis appointment was just after the person who had an interview in room R4, who
had an appointment just after Chirag.
(iv) Three of the four people were: (1)Deepak, (2) the one with interview in room R1, and (3)
the one who was interviewed at 1 p.m.(this is a crazy one!
(v) Interview times were 11 a.m., 12 noon, 1 p.m., and 2 p.m.(This could be a row heading.
(vi) Sharma, Narurkar, Joshi and Zaidi were interviewers and Teena, Bimal, Chirag and
Deepak were the interviewees.

Q1. Sharmas appointment is with


(a) Teena
(b) Bimal
(c) Chirag
(d) Deepak

Q2. Deepaks appointment was in room

(a) R1
(b)R3
(c)R2
(d) R4

Q3. Mr. Zaidi interviewed


(a) Deepak
(b) Teena
(c) Bimal
(d) Chirag

Q4. Who interviewed at 2 p.m.?


(a) Mr. Joshi
(b) Mr. Zaidi
(c) Mr. Narurkar
(d) Mr. Sharma

Q5. Which candidate was interviewed last?


(a) Deepak
(b) Teena
(c) Bimal
(d) Chirag

The option in italics is the correct answer. How many did you get right?! And a sample
table of the
SOLUTION:

Interviewee Interviewer 11 a.m. 12 p.m. 1 p.m. 2 p.m.


Chirag Sharma R1 X Definite X X
Bimal Zaidi R2 Definite X X X
Deepak Joshi R3 X X X Definite
Teena Narurkar R4 X X Definite X
SET 7
Read the following information carefully and answer the questions which follow.

Five friends A, B, C, D, and E are working in 5 different departments M, N, O, P and Q and


they earn different salaries i.e. 10,000, 15,000, 20,000, 25,000 and 30,000 and they all are of different
ages i.e. 24, 26, 28, 30 and 32 years. These all informations are not necessarily in the same order.

B works in department M and earns more than 20,000. Person who is 28 years old works in
department Q. 32 years old person earns at least 20,000.

The person who is 26 years old earns 25,000. A earns 15,000, but does not work in
department N or P.
Person who is 30 years old earns highest salary but does not work in department M and N. E
does not work in department P or Q, and his age is not 32. The salary of D is less than 20,000.

1. Who works in department N?


a) B b) C c) D
d) Cant be determined e) None of these

2. If the name of the person represents its salary then which of the following is true?
a) A + B = C b) C + D = B c) D + E = B
d) Cant be determined e) None of these

3. Which of the following combination is definitely true?


a) B-30 years-M-30,000
b) D-24 years-N-10,000
c) A-24 years-P-15,000
d) E-30 years-O-30,000
e) None of these

4. The person whose age is 30 works in which department?


a) M b) N c) O
d) P e) None of these

Answers:
1. Option D
2. Option E
3. Option D
4. Option C

SET 8
Directions (1-5): Read the following information carefully and answer the questions given below
it

Six exams Maths, science, History, Economics, English and Hindi are to be scheduled starting from
2nd March and ending on 8th march wit Sunday being an off day, not
necessarily in the same order. Each of the exam has different time duration: 40 mins, 50 mins, 60
mins, 75 mins, 90 mins and 100 mins, again not necessarily in the same order.
8th march is not sunday and an exam of 40 mins is scheduled on that day. Maths exam is for less than
60 mins and is scheduled immediately before English exam. There are two exams scheduled between
Hindi exam which is for 100 mins and History exam which is for 60 mins. English exam is before
Sunday and there are two days between sunday and maths exam. Economics exam which is for 75
mins is not scheduled on 2ndmarch. The exam schedules on saturday is of 100 mins.

1. How many exams are scheduled before sunday?


(1) Two (2) One
(3) Five (4) Three
(5) None of these

2. Which of the following combinations of exam - Day - Time Duration is correct ?


(1) English - Wednesday - 75 mins (2) Maths - Thursday - 50 mins
(3) History - Thursday 60 mins (4) Hindi - Tuesday - 100 mins
(5) None is correct

3. What is the time duration of science exam?


(l) 90 mins (2) 75 mins
(3) 50 mins (4) 40 mins
(5) None of these

4. On which day is Economics exam scheduled?


(1) Monday (2) Saturday
(3) Tuesday (4) Friday
(5) Cannot be determined

5. Which day is sunday?


(1) 3rd march (2) 2nd march
(3) 5th march (4) 6th march
(5) Cannot be determined

Solutions

Solutions (1-5):

Date Day Exam Time Duration


2nd
March Wednesday History 60 mins
3rd
March Thursday Maths 50 mins
4th
March Friday English 90 mins
5th
March Saturday Hindi 100 mins
6th
March Sunday Off Off
7th
March Monday Economics 75 mins
8th
March Tuesday Science 40 mins

1. (5)
2. (2)
3. (4)
4. (1)
5. (4)

SET 9
Directions (6-12): Each of these questions are based on the information given below:

P, Q, R, S, W, X, Y, Z are sitting around a circle facing centre but not necessarily in the same order.
The husband of Z is sitting second to the right of Q who is sitting between two males. X sits second to
the left of the daughter of S. X is the sister of Y. X is not an immediate neighbour of Z's
husband. Only one person sits between P and X. P is the father of Y. S who is brother of Z sits to the
immediate left of his mother. Only one person sits between Z's mother and W. Only one person sits
between Z and Y. Y is mother of R. Y is not an immediate neighbour of W.

6. Which of following is true with respect to the given seating arrangement?


(1) R is the cousin of W
(2) Z and Z's husband are immediate neighbour of each other.
(3) No female is an immediate neighbour of R.
(4) Z sits third to the left of her daughter.
(5) Q is the mother of Z.

7. What is the position of P with respect to his grandchild?


(1) immediate right (2) third to the right
(3) third to the left (4) Second to the right
(5) fourth to the right
8. Four of the following five are alike in a certain way. Which one does not belong to that group?
(1) X (2) R
(3) W (4) Z
(5) Y

9. What is position of P with respect to his mother-in-law?


(1) Immediate right (2) third to the right
(3) third to the left (4) second to the right
(5) None of these

10. How many people sits between Y and her uncle?


(1)One (2) Two
(3) Three (4) Four
(5) Five
11. . Who amongst the following is S's daughter?
(1) Q (2) R
(3) W (4) Y
(5) Z

12. Who sits to the immediate left of R?


(1) X's grandmother (2) Y's son
(3) S's mother-in-law (4) P
(5) Y

Solutions (6-12):
6. (5)
7. (1)
8. (2)
9. (4)
10. (3)
11. (3)
12. (1)

SET 10
Direction (1 5): Read the following information and answer the questions that follow:

(1) There are seven persons A, B, C, D, E, F and G based in Delhi. Each of them is from a different
state, has a different profession, and plays a different instrument.
(2) C, a doctor, is from Bihar.
(3) E and F play mandolin and violin, though not necessarily in that order.
(4) A is not from Kerala.
(5) The person from Kerala is an engineer and plays guitar.
(6) The lawyer plays sitar.
(7) The businessman from UP plays violin.
(8) The teacher and the cricketer play flute and piano, though not necessarily in that order.
(9) F is a pilot.
(10) The Maharashtrian is a teacher.
(11) The Gujarati plays piano.
(12) G, a Punjabi, does not play sarod.
(13) B is a cricketer.

1. Which state does A belong to?


A) Gujarat
B) Kerala
C) Maharashtra
D) Cant say
E) None of these

2. Which instrument does B play?


A) flute
B) piano
C) sarod
D) Cant say
E) sitar

3. Which instrument does C play?


A) mandolin
B) sitar
C) violin
D) None of these
E) sarod

4. What is Ds profession?
A) Engineer
B) Lawyer
C) Teacher
D) Cant say
E) Cricketer

5. Which state does E belong to?


A) Gujarat
B) Kerala
C) UP
D) Cant say
E) none of these

SET 11
Directions (6 -10): Study the following information and answer the given question.

Eight friendsJ, K, L, M, N, O, P and Q live on eight different floors of a building but not necessarily
in the same order. The lowermost floor of the building is numbered one, the one above that is
numbered two and so on till the topmost floor is numbered eight. J lives on floor numbered 4. Only
one person lives between J and L. O lives on the floor immediately below L. Only one person lives
between O and P. P lives above O. O lives on floor numbered 5. Only one person live between K and
N, M lives on an even numbered floor but does not live on floor numbered 8. K lives below Qs
floor.

6. Four of the following five are alike as per the given arrangement and thus form a group. Which of
the following does not belong to that group?
(A) PO
(B) QL
(C) MJ
(D) LJ
(E) MN

7. Who lives exactly between the floors on which L and J live?


(A) K
(B) P
(C) O
(D) N
(E) M

8. Who amongst the following lives on floor numbered 2?


(A) M
(B) L
(C) O
(D) Q
(E) P

9. Which of the following is true about M as per given arrangement?


(A) M lives on the lowermost floor.
(B) K lives on one of the floor below M.
(C) All the given statements are true.
(D) Only two people live between M and O.
(E) M lives on an odd numbered floor.

10. If L and Q interchange their places and so do M and J, who amongst the following will live
between the floors on which M and P live, as per the given arrangement?
(A) Only Q
(B) No one
(C) Only L
(D) Both P and L
(E) Both Q and O

Answers
1. c
2. b
3. e
4. a
5. c
6. e
7. c
8. a
9. d
10.e

SET 12
Directions (Questions 1 - 5): Study the following information carefully to answer the given
questions:

Madan and Rohit are in the same team of hockey. Parth defeated Rohit in badminton but lost to
Sachin in tennis. Nitin teams with Sagar in football and with Sachin in hockey. Rohit defeated Sachin
in chess. Those who play cricket do not play badminton, volleyball or tennis. Madan and Parth are in
opposite teams of basketball. Nitin represents his state in cricket while Sagar does so at the district
level. Boys who play chess do not play football, basketball or volleyball. Madan and Parth are
together in the volleyball team. Boys who play football also play hockey.

1. Name the boys who do not play football.


(a) Sachin, Nitin
(b) Rohit, Sagar
(c) Rohit, Sachin
(d) Rohit, Nitin
(e) None of these

2. Who plays both hockey and tennis?


(a) Sachin
(b) Rohit
(c) Nitin
(d) Parth
(e) None of these

3. Which is the most popular game with this group?


(a) Cricket
(b) Hockey
(c) Football
(d) Badminton
(e) None of these

4. Who plays the largest number of games?


(a) Sagar
(b) Rohit
(c) Parth
(d) Nitin
(e) None of these

5. Which boy plays both badminton and hockey?


(a) Sachin
(b) Rohit
(c) Nitin
(d) Parth
(e) None of these

Answers

Madan Rohit Parth Sachin Nitin Sagar


Hockey Y Y Y Y Y
Badminton Y Y N N
Tennis Y Y N N
Chess Y Y
Football N N Y Y
Basketball Y N Y N
Volleyball Y N Y N N N
Cricket Y Y
1. c
2. a
3. b
4. c
5. b

SET 13
Directions (Questions 6 - 10): Study the following information carefully and answer the questions
given below:

Seven boys A, D, Y, U, P, Q and J live in three different buildings Ashiana, Top-view and Ridge.
Each of them is flying kites of different colours i.e. red, green, blue white, black, yellow and pink, not
necessarily in that order. Not more than three or less than two stay in any of the buildings. Q is flying
a pink and does not live in Ridge building. U does not live in the same building as A or P and is flying
a yellow coloured kite. D lives in Ridge building with only one more person and is flying a green kite.
None in the Top-view building flies a white kite. P does not fly a blue kite.

6. Who live in Ridge building?


(a) D, U
(b) D, A, P
(c) Y, A, P
(d) A, P
(e) None of these

7. Who is flying the blue kite?


(a) A
(b) J
(c) P
(d) Data inadequate
(e) None of these
8. Who flies the red kite?
(a) A
(b) J
(c) P
(d) Data inadequate
(e) None of these

9. Who stay in Top-view building?


(a) Y, J, P
(b) A, P
(c) A, P, D
(d) Y, U, J
(e) None of these

10. What coloured kite is J flying?


(a) Blue
(b) White
(c) Black
(d) Data inadequate
(e) None of these

Solution

Boy Building Kite


A Top-View Blue
D Ridge Green
Y Top-View Black
U Ridge Yellow
P Top-View Red
Q Ashiana Pink
J Ashiana White

6. a
7. a
8. c
9. e
10. b

SET 14
Direction (1-5): Read the following information carefully and answer the questions given below:

Six persons A, B, C, D, E and F are sitting around a round table facing towards the centre of the table
in a restaurant. They have ordered for different items (Pizza, Strawberry, Vanilla, Burger, Pastries and
Patties) as their lunch. They are wearing T-shirts of different colours, i.e. white, black, green, red,
yellow and blue. Order of items for the lunch and colours of T-shirts are not necessarily according to
the order of their names.

I. The persons who have ordered for Pizza, Vanilla and Pastries are neither in white T-shirt
nor in black.
II. The persons who are in green and yellow T-shirts have neither ordered for Pizza nor for
Vanilla.
III. A is neither in white T-shirt nor on the immediate left of the person who has ordered for
Burger.
IV. The only person who is between E and F eats Strawberry. The person who is on the left
side of the person in white T-shirt does not eat Patties.
V. D has ordered Burger and the colour of his T-shirt is green. He is facing the person who
has ordered for Strawberry.
VI. One who has ordered for Pizza is seated opposite to the person wearing blue T-shirt,
while the person whose T-shirt is of green colour is on the left of the person who has
ordered for Pastries.
VII. One who has ordered for Patties is on the immediate right of the person in white T-shirt
but on the immediate left of the person who has ordered for Vanilla.
VIII. C has not ordered for Vanilla while F has not ordered for Pizza.

1.Who among the following is in white T-shirt?


a) A
b) B
c) C
d) E
e) None of these

2.The only person, who is between E and D, is wearing T-shirt of the colour
a) red
b) blue
c) black
d) yellow
e) None of these

3.Who among the following has ordered for Pastries?


a) A
b) B
c) C
d) E
e) None of these

4.Which of the following is correctly matched?


a) A - Yellow - Burger
b) B - Red - Vanilla
c) E - Red - Pizza
d) F - Black - Pastries
e) None of these

5.The colour of the T-shirt of the person, who has ordered for Patties, is
a) red
b) yellow
c) blue
d) black
e) None of these

Answers

1. C
2. D
3. A
4. C
5. D

SET 15
Directions (Questions 6 to 8): Read the following information carefully and answer the questions
given below:

(i) There are five types of cards viz. A, B, C, D and E. There are three cards of each type. These
are to be inserted in envelopes of three colours Red, Yellow and Brown. There are five
envelopes of each colour.
(ii) B, D and E type cards are inserted in red envelopes. A, B and C type cards are to be inserted
in yellow envelopes and C, D and E type cards are to be inserted in brown envelopes.
(iii) Two cards each of B and D type are inserted in red envelopes.

6.Which of the following combinations of types of cards and the number of cards and colour of
envelope is definitely correct?
a) A-2, B-2, C-1 : Yellow
b) C-2, D-1, E-2 : Brown
c) C-1, D-2, E-2 : Brown
d) B-2, D-2, A-1 : Red
e) None of these

7.Which of the following combinations of colour of the envelope and the number of cards is definitely
correct in respect of E-type cards?
a) Red - 1, Yellow -2
b) Yellow 1, Brown - 2
c) Red - 2, Brown - 1
d) Red - 2, Yellow -1
e) None of these

8.Which of the following combinations of the type of cards and the number of cards is definitely
correct in respect of yellow coloured envelopes?
a) A - 2, E - 1, D - 2
b) A - 2, B - 1, C - 2
c) A - 3, B - 1, C - 1
d) B - 1, C - 2, D - 2
e) None of these

Answers
6. b
7. e
8. c

Directions (Questions 9 to 10): Read the following information carefully and answer the questions
given below:
Three persons A, B and C wore shirts of black, blue and orange colours (not necessarily in that order)
and pants of green, yellow and orange colours (not necessarily in that order). No person wore pant and
shirt of the same colour. Further, it is given that

1) A did not wear shirt of black colour.


2) B did not wear shirt of blue colour.
3) C did not wear shirt of orange colour.
4) A did not wear pant of green colour.
5) B wore pant of orange colour.

9.What were the colours of pant and shirt worn by C, respectively?


a) Yellow and Black
b) Yellow and Blue
c) Green and Blue
d) Orange and Black
e) None of these

10.Who did not wear the orange colour?


a) A
b) B
c) C
d) A & C
e) None of these

Answers

9. c
10. c

SET 16
Direction (1-5): Read the following information carefully and answer the questions given below it :
There are six persons A, B, C, D, E and F in school. Each of the teachers teaches two subjects, one
compulsory subject and the other optional subject. Ds optional subject was History while three others
have it as compulsory subject. E and F have Physics as one of their subjects. Fs compulsory subject
is Mathematics which is an optional subject of both C and E. History and English are As subjects but
in terms of compulsory and optional subjects, they are just reverse of those of Ds. Chemistry is an
optional subject of only one of them. The only female teacher in the school has English as her
compulsory subject.

1.What is Cs compulsory subject ?


(a) History
(b) Physics
(c) Chemistry
(d) English
(e) Mathematics

2.Who is a female member in the group ?


(a) A
(b) B
(c) C
(d) D
(e) E

3.Which of the following has same compulsory and optional subjects as those of Fs ?
(a) D
(b) B
(c) A
(d) C
(e) None of these

4.Disregarding which is the compulsory and which is the optional subject, who has the same two
subject combination as F ?
(a) A
(b) B
(c) E
(d) D
(e) None of these

5.Which of the following groups has History as the compulsory subject ?


(a) A, C, D
(b) B, C, D
(c) C, D
(d) A, B, C
(e) A, D

Answers

1. a
2. d
3. e
4. c
5. d

SET 17
Direction (6-10): Read the following information carefully and answer the questions that follow:

(i) Five friends P, Q, R, S and T travelled to five different cities of Chennai, Kolkata, Delhi,
Bangalore and Hyderabad by different modes of transport of Bus,Train, Aeroplane, Car
and Boat from Mumbai.
(ii) The person who travelled to Delhi did not travel by boat
(iii) R went to Bangalore by car and Q went to Kolkata by aeroplane
(iv) S travelled by boat whereas T travelled by train
(v) Mumbai is not connected by bus to Delhi and Chennai

6.Which of the following combinations of person and mode is not correct ?


(a) P Bus
(b) Q Aeroplane
(c) R Car
(d) S Boat
(e) T Aeroplane

7.Which of the following combinations is true for S ?


(a) Delhi - Bus
(b) Chennai - Bus
(c) Chennai - Boat
(d) Data inadequate
(e) None of these

8.Which of the following combinations of place and mode is not correct ?


(a) Delhi - Bus
(b) Kolkata - Aeroplane
(c) Bangalore - Car
(d) Chennai - Boat
(e) Hyderabad Bus

9.The person travelling to Delhi went by which of the following modes ?


(a) Bus
(b) Train
(c) Aeroplane
(d) Car
(e) Boat

10.Who among the following travelled to Delhi ?


(a) R
(b) S
(c) T
(d) Data inadequate
(e) None of these

Solutions
6. E
7. c
8. a
9. b
10.c

SET 18
Directions (Q. 1 - 5): Study the information carefully and answer the following questions:

A building has eight floors from 1 to 8 in such a way that ground floor is numbered 1, the above floor
is numbered 2 and so on. The top most floor is numbered 8. Eight persons G, P, M, R, Q, A, D and C
are staying each on one of these floors. A is staying on floor 6. There is a gap of three persons
between G and R. C is staying on the topmost floor. Neither P nor M is staying on the floor 5. Q is
staying middle of P and M. R is staying on the floor 1. M is not an immediate neighbor of G.

Q1. Who is staying on the floor no. 7?


1) A
2) P
3) D
4) R
5) M

Q2. How many persons are staying between D and P?


1) Three
2) Four
3) Two
4) One
5) None of these

Q3. Who is staying on the second floor?


1) Q
2) M
3) P
4) A
5) Q or M

Q4. If all persons are arranged in alphabetical order such as A occupies floor 1 then who will occupy
floor 5?
1) G
2) Q
3) R
4) M
5) P

Q5. Immediate neighbors of the person who is staying in floor 3?


1) R and P
2) A and P
3) G and M
4) P and M
5) D and G

Solutions
1-3
2-3
3-2
4-4
5-4

Floor No. Persons


8 C
7 D
6 A
5 G
4 P
3 Q
2 M
1 R

SET 19
Directions (Q. 6 - 10): Study the information carefully and answer the following questions:

Eight friends Romil, Ramesh, Rakesh, Rohit, Rahul, Abhijit, Abhishek and Anil are sitting around
a circular table, not necessarily in the same order. Four of them are facing inside others are facing
outside. They are belong to eight different cities Bhopal, Patna, Kolkata, Delhi, Gwalior, Bengaluru,
Chennai and Rajkot, but not necessarily in the same order.

Abhijit faces the Centre and sits third to the right of Rakesh. Rohit belongs to Kolkata and faces the
person who belongs to Bengaluru. Abhishesk sits third to the right of Ramesh, who stays in Bhopal.
The persons who belong to Delhi and Gwalior are facing to the same direction (inside or outside).
Rahul is sitting between the person who belongs to Kolkata and the one from Rajkot respectively.
Romil belongs to Gwalior and Rakesh belongs to Patna. The person who belongs to Chennai is facing
outward and immediate neighbor of Rajkot. Anil is immediate neighbor of the persons who belong to
Gwalior and Chennai. Rahul is immediate left of Rohit.

Q6. Who belongs to Bengaluru?


1) Romil
2) Rohit
3) Anil
4) Abhishek
5) Rahul

Q7. Immediate neighbors of Romil?


1) Romesh and Rakesh
2) Rahul and Rohit
3) Anil and Ramesh
4) Abhishek and Rahul
5) Rohit and Abhijeet

Q8. Which of the following pair is true?


1) Romil Bhopal
2) Anil Bengaluru
3) Rohit Delhi
4) Abhijeet Delhi
5) Abhishek Patna

Q9. Position of Ramesh with respect to Rahul?


1) Third to the right
2) Fourth to the left
3) Fifth to the right
4) Second to the right
5) Third to the left

Q10. If Rohit and Anil interchange their positions and similarly Abhijit and Rakesh interchange their
positions then what is the position of Romil with respect to Rakesh?
1) Third to the left
2) Third to the right
3) Second to the left
4) Immediate left
5) None of these

Solutions
6) 3
7) 3
8) 2
9) 5
10) 1
SET 20
Directions for questions: These questions are based on the information given below.

Eight members P, Q, R, S, T, U, V, W of a family are sitting around a circular table. The family
members belong to three generations and each couple has at least one daughter and one son.
Q, the grandfather of U, has more number of children than the number of grandchildren. S is opposite
his mother T, who is adjacent to her granddaughter. V is to the immediate left of her brother-in-law.
Ps daughter R, is to the immediate right of her aunt W. P is third to the right of his sister.

1. What is the position of R with respect to T?


1) Immediate left
2) Immediate right
3) Second to the left
4) Second to the right
5) Either (1) or (2)

2. How many couples are there in the family?


1) One
2) Two
3) Three
4) Four
5) Cannot be determined

3. What is the position of Q with respect to S?


1) Second to the left
2) Immediate right
3) Second to the right
4) Immediate left
5) Cannot be determined

4. How is S related to U?
1) Uncle
2) Mother
3) Father
4) Aunt
5) Grand mother

5. Which of the following pairs are not siblings?


1) SW
2) PS
3) UR
4) WP
5) PV

Solutions
1) 1
2) 2
3) 5
4) 1
5) 5

SET 21
Directions (1-5) Study the following information and answer the following questions.

A, B, C, D, E, G, and I are seven friends who study in three different standards namely 5th, 6th, 7th
such that not less than two friends study in the same standard. Each friend also has a
differentfavourite subject namely History, Civics, English, Marathi, Hindi, Maths and Economics but
not necessarily in the same order.

A like Maths and studies in 5th standard with only one other friends who likes Marathi. I studies with
two other friends. Both the friends who study with I like language (here languages include only Hindi,
Marathi and English).

D studies in 6th standard with only one person and does not like Civics. E studies with only one
friend. The one who likes history does not study in 5th or 6th standard. E does not like languages. C
does not like English, Hindi or Civics.

1. Which combination represents Es favourite subject and the standard in which he studies?
(1) Civics and 7th
(2) Economics and 5th
(3) Civics and 6th
(4) History and 7th
(5) Economics and 7th

2. Which of the following is Is favourite subject?


(1) History
(2) Civics
(3) Marathi
(4) Either English or Marathi
(5) Either English or Hindi

3. Who amongst the following studies in 7th standard?


(1) G
(2) C
(3) E
(4) D
(5) Either D or B

4. Which of the combination is definitely correct?


(1) I and Hindi
(2) G and English
(3) C and Marathi
(4) B and Hindi
(5) E and Economics

5. Which of the following subject does G like?


(1) Either Maths or Marathi
(2) Either Hindi or English
(3) Either Hindi or Civics
(4) Either Hindi or Marathi
(5) Either Civics or Economics

Answers:
1 (3)
2 (1)
3 (1)
4 (3)
5 (2)

SET 22
There are 5 houses in 5 different colours. In each house lives a person of a different nationality. The 5
owners drink a certain type of beverage, smoke a certain brand of cigar, and keep a certain pet.
Using the clues below can you determine who owns the fish?

a) The Brit lives in a red house.


b) The Swede keeps dogs as pets.
c) The Dane drinks tea.
d) The green house is on the immediate left of the white house.
e) The green house owner drinks coffee.
f) The person who smokes Pall Mall rears birds.
g) The owner of the yellow house smokes Dunhill.
h) The man living in the house right in the middle drinks milk.
i) The Norwegian lives in the first house.
j) The man who smokes Blend lives next door to the one who keeps cats.
k) The man who keeps horses lives next door to the man who smokes Dunhill.
l) The owner who smokes Blue Master drinks chocolate.
m) The German smokes Prince.
n) The Norwegian lives next to the blue house.
o) The man who smokes Blend has a neighbor who drinks water.

Answer:
This puzzle is usually attributed to Einstein, who may or may not have written it.

The German owns the fish and the table below details the full answer:

Nationality Norweg Dane Brit German Swede


Color Yellow Blue Red Green White
Beverage Water Tea Milk Coffee Chocolate
Smokes Dunhill Blend Pall Mall Prince Blue Master
Pet Cats Horses Birds Fish Dogs

If in a cricket one day match, there were no wide balls, no wides, no extras and no overthrows.
What is the maximum number of runs that a batsman can score in an ideal case scenario?

Solution
1653

Explanation:

For an ideal case, the batsman will hit a six on each ball. But if he hits six on the last ball of the over,
the strike will change in the next over. Thus, the best he can do in the last ball is run 3 runs so that he
retains the strike even in the next over. Thus the total runs that he can score in each over:
6 * 5 + 3 = 33

But this will have to go like it only till the 49th over. In the last over, he can hit a six in the last ball as
well as that will be the last ball of the match.
Thus runs for the last over will be 6 * 6 = 36.
Hence the maximum runs = 33 * 49 + 36 = 1653

SET 23
Study the following information carefully to answer the questions given below.

A, B, C, D, E, F and G are 7 friends left for 7 different cities Delhi, Chennai, Hyderabad,
Bangalore, Kolkata, Chandigarh and Jaipur, each one on a different day of the week. C left for Jaipur
on Monday. On the last day of the week, the person left for Bangalore. E left the next day of A who
left for Chandigarh and on the previous day of G. D left for Kolkata on Friday. B didnt leave for
either Hyderabad or Bangalore and G left for Delhi.

1. On which day of the week did B leave ?


A)Sunday
B)Saturday
C)Monday
D)Tueday
E)None of these

2. Who left for Bangalore ?


A)E
B)B
C)G
D)F
E)None of these

3. Who left on Tuesday ?


A)A
B)C
C)G
D)F
E)None of these

4. On which day of the week did E leave ?


A)Friday
B)Sunday
C)Wednesday
D)Thursday
E)None of these

5. Which of the following Combination is not right ?


A)C Jaipur
B)F Bangalore
C)A Chandigarh
D)E Kolkata
E)None of these

Solutions

1. B
2. D
3. A
4. C
5. D

SET 24
Study the following information carefully to answer the questions given below.

P, Q, R, S, T, U and V are 7 friends who travel to college everyday by a particular train which
stops at 5 stations 1, 2, 3, 4 and 5 respectively after it leaves base stations. 3 among them get in the
train at the base station. S gets down at the next station at which U gets down. Q gets in with 2
persons and does not get down with either P or T. V alone gets in at station 3 and gets down with R
after 1 station. P travels between only 2 stations and gets down at station 5. None of them gets in at
station 2. R gets in with U but does not get in with either Q or S. T gets in with 2 others and gets down
alone after S. Q and S going to same college and they get down together at station 3. None of them
gets down at station 1.
1. At which station does T get down ?
A)Station 2
B)Station 4
C)Station 3
D)Station 5
E)None of these

2. At which station does R, U get in ?


A)Base station
B)Station 2
C)Station 1
D)Station 3
E)None of these

3. After how many station does Q get down ?


A)4
B)3
C)1
D)2
E)None of these

4. At which of the following station does Q and T get in ?


A)Base Station
B)Station 1
C) Station 2
D) Station 3
E)None of these

5. Which of the following is correct ?


A)T gets in at the base station
B)R gets in at the Station 3
C)V gets down at Station 5
D)U gets down at Station 2
E)None of these

Answers:

1. B
2. C
3. D
4. A
5. A

You might also like